LSAT and Law School Admissions Forum

Get expert LSAT preparation and law school admissions advice from PowerScore Test Preparation.

 Administrator
PowerScore Staff
  • PowerScore Staff
  • Posts: 8917
  • Joined: Feb 02, 2011
|
#23433
Complete Question Explanation

Weaken-#%. The correct answer choice is (E)

This stimulus consists entirely of information, but you can draw a conclusion. Since nutritionists recommend that people eat more fiber, and the fiber-supplement pill advertisements focus on the pill's fiber content, you can conclude that the advertisement is meant to convince people that the pill is a good way to get fiber.

The question stem asks you to make the advertisement misleading. Essentially, you must weaken the idea that the advertisement is legitimate. You should focus on the "44 percent." Even though a significant portion of the pill is made of fiber, the pill itself could be very small. If the pill is too small, it might not be a very good way to get fiber. That is a "% vs. #" concept, because you are required to realize that if the pill is too small-- a numbers concept--, the percentage of fiber it contains might mean nothing.

Also, some people will incorrectly interpret the advertisement to mean that the pill provides 44% of daily fiber requirements. Since the advertisement could have made the issue a bit clearer, it is quite possible that the advertisement was intended to trick people who are not good at reading sentences for their actual mathematical meanings.

Answer choice (A): The issue is whether this supplement is good, not whether others are better.

Answer choice (B): We are not interested in the nutritionists' reasoning behind why fiber is beneficial; we are concerned with whether this pill provides significant fiber.

Answer choice (C): It is unreasonable to believe that fiber could be addictive in the same manner that painkillers or sleeping pills are, so this choice is without relevance. Furthermore, the issue is whether people will get enough fiber, not whether they will get too much.

Answer choice (D): The recommended dosage proves nothing immediately about whether the pills contribute a significant supplement, so this choice is irrelevant, and incorrect. If the recommended dosage was unreasonably high, we might be getting closer to suggesting that it would be overly inconvenient or expensive to use the pills. 3 pills is well within normalcy.

Answer choice (E): This is the correct answer choice. These numbers mean that the pills contain 1.1% to 1.67% of the recommended daily intake of fiber. That makes it highly likely that the pills are not an effective supplement, because quite a number of pills would be required per day if a person's diet leaves out even 10% or 15% of required fiber. If a person wanted to get all his fiber from these pills, he'd have to consume 60 to 90 pills. Ouch!

Furthermore, this answer choice shows that the 44% can be misleading. May people, when they see "44 percent fiber," will assume that the label means "44 percent of daily fiber requirement," even though that is clearly not the meaning. However, answer choice (E) rules that possibility out, because each pill contributes less than 2% of necessary fiber. Since we know that a great many people might misinterpret the statistic, this new information demonstrates that the advertisement might have been constructed to take advantage of those people.
 yrivers
  • Posts: 68
  • Joined: Mar 15, 2017
|
#33480
Could you please expand on why choice D is incorrect? Immediately reading that, I would think that approximately 3 of these pills are required. Is it incorrect because there's no additional information to decide what 44% actually means? Bit confused.
 Adam Tyson
PowerScore Staff
  • PowerScore Staff
  • Posts: 5153
  • Joined: Apr 14, 2011
|
#33481
Compare the numbers in D to the numbers in E, and you'll see that E presents a much bigger problem - 3 pills in D vs 60-90 pills in E. Also, in D we are only looking at what the pill maker recommends, but in E we are looking at what nutritionists recommend, which is more relevant to the question of whether the ad is misleading.

In fact, if the pill gave you 44% of the daily recommended dose (one possible misinterpretation of the label that the pill makers might have been going for), then taking three pills would be one way to get your daily dose. That would be a good thing if the numbers were right. Again, though, look at 3 pills vs 90 pills and E is much, much more troubling.

Since we are tasked with picking the best answer of the five presented, and since E is better (more problematic, in this case) than D, we go with E.

Remember, it's not about picking the right answer - it's about picking the BEST answer!
 yrivers
  • Posts: 68
  • Joined: Mar 15, 2017
|
#33487
I understand now, thank you so much!

Get the most out of your LSAT Prep Plus subscription.

Analyze and track your performance with our Testing and Analytics Package.